LSAT 55 – Section 3 – Question 10

You need a full course to see this video. Enroll now and get started in less than a minute.

Target time: 1:03

This is question data from the 7Sage LSAT Scorer. You can score your LSATs, track your results, and analyze your performance with pretty charts and vital statistics - all with a Free Account ← sign up in less than 10 seconds

Question
QuickView
Type Tags Answer
Choices
Curve Question
Difficulty
Psg/Game/S
Difficulty
Explanation
PT55 S3 Q10
+LR
Sufficient assumption +SA
A
2%
154
B
1%
154
C
1%
155
D
81%
163
E
15%
160
125
141
156
+Easier 144.364 +SubsectionEasier

The Question Stem reads: The lawyer's conclusion follows logically if which one of the following is assumed? This is a Sufficient Assumption question.

The lawyer begins by describing how this witness was present at the restaurant when the lawyer's famous client was assaulted. However, the witness claims to recognize the assailant but not the famous client. The lawyer concludes the witness's testimony should be excluded. We can break down the argument to read:

P1: Witness claims to recognize the assailant

P2: Witness claims not to recognize the victim (Famous client)

________________

C: Exclude witness testimony

In the CC, we discussed that elements of the conclusion must be in the premises. Nowhere in the premises do we see a claim about what kind of testimony should not be included. We need a conditional that brings us to "exclude testimony," so let's make that our necessary condition: (__) -> Exclude testimony.

Now it will be hard to anticipate what sufficient condition the AC will use. They could use P1 or P2 or some combination of both. When we screen these answer choices, the first order of business will be to make sure that the necessary condition is: "exclude testimony" Then we will check the sufficient condition to ensure it gets triggered by the information in the stimulus. Let's go.

Answer Choice (A) has the necessary condition "then the witness's testimony should be included." Without looking at the rest of the (A), we can eliminate it because it takes us to "include" when we want to go to "exclude." If you picked (A), you likely assumed that "claims recognize both parties -> include" implied that "/(claims recognize both parties) -> exclude," which is a logical fallacy. Remember: a->b does not imply /a->/b.

Answer Choice (B) is arbitrary. Why would the fact that other witnesses can identify the client mean we should exclude the witness from the stimulus? Nothing. Eliminate and move on.

Answer Choice (C) is also arbitrary because it does not bring us to a conclusion "exclude testimony." As a side note, whether or not we can know if the witness actually recognized the assailant is irrelevant. Notice how the premises only take into account who the victim claims to recognize. The lawyer's argument is going to rely on the witness's claims, not what actually is the case.

Correct Answer Choice (D) gets us to where we need to go. If we take the contrapositive of (D), we get: "/(claims to recognize both parties in assault) -> exclude." The necessary condition is exactly what we discussed. The sufficient condition is great. The witness did not claim to recognize both parties in the assault. The witness claims to recognize only the assailant. So the conditional triggers and delivers us to the conclusion that the witness's testimony should be excluded.

Answer Choice (E) is a popular wrong answer. If you picked (E), you likely inferred that the witness was lying about not recognizing the famous victim. First, that is not an inference you can make. Just because it is unlikely that someone wouldn't recognize the famous client, that does not mean it is impossible that someone would fail to recognize the client. Second, even if we could infer that the witness was lying, it wouldn't help us. What does lying have to do with excluding testimony? If you answered that "liars testimony should be excluded," you've proved that (E) by itself is insufficient to draw the lawyer's argument.

Take PrepTest

Review Results

Leave a Reply